PLEASE help me with this question! No nonsense answers please. This is really urgent.

PLEASE Help Me With This Question! No Nonsense Answers Please. This Is Really Urgent.

Answers

Answer 1

Answer:

32

Step-by-step explanation:

This is because you would want the feet and the radium kinda meeting up in a way and would need to make sure you can fill it in with th e drive way as it will be on a horizontal base so you would need X2 of the driveway to make it into a standing up horizontal base

Hope this helps

If this seems incorrect please comment with the answer that it should be and I shall change my answer thanks you :)


Related Questions

Lucy reads 450 words in 3 minutes.
This is an equation that can be used to
find w, the number of words Lucy can
read in 20 minutes if she continues to
read at the same rate?

Answers

Answer:

3000 words

Step-by-step explanation:

450 words in 3 minutes is 150 words in 1 minute. 20 minutes = 1 times 20.          

1 minute = 150 words so 150 words times 20 minutes = 3000 words.

ASAP PLZ ANSWER!!! Can you tell me step by step to this question 8,595 ÷ 24?

Answers

Answer:

358 and remainder of 3

Step-by-step explanation:

1. Divide it like any other problem

24 goes into 85, 3 times with 13 left overBring down the 9 and 24 goes into 139, 5 times with 19 left overThen bring down the 5 and 24 goes inside 195, 8 times with 3 left overSo your remainder would be 3

Hope this helps

Show all work to solve the equation for x. If a solution is extraneous, be sure to identify it in your final answer.

square root of the quantity x minus 3 end quantity plus 5 equals x

Answers

Answer:

Step-by-step explanation:

[tex]\sqrt{x-3} +5=x\\\sqrt{x-3} =x-5\\squaring ~both~sides\\x-3=x^2-10x+25\\x^2-10x-x+25+3=0\\x^2-11x+28=0\\x^2-7x-4x+28=0\\x(x-7)-4(x-7)=0\\(x-7)(x-4)=0\\x=7,4[/tex]

put x=7 in the given equation

[tex]\sqrt{7-3} +5=7\\\sqrt{4} +5=7\\2+5=7\\7=7[/tex]

which is true .

∴ x=7 is a solution of the given eq.

now put x=4 in the given eq.

[tex]\sqrt{4-3} +5=7\\1+5=7\\6=7\\[/tex]

which is not true.

∴x=4 is an extraneous solution.

How many solutions does this system of inequalities have graphed below

Answers

Answer:

There is only one solution set.

Step-by-step explanation:

The set can contain any number of solutions or none.

This is all I know, hope it helps.

Find f(x) and g(x) so the function can be expressed as y = f(g(x)). (1 point) [tex]y=\frac{7}{x^{2} } +10[/tex]

Answers

Answer:

The functions are [tex]f(x) = 7\cdot x+10[/tex] and [tex]g(x) = \frac{1}{x^{2}}[/tex], respectively.

Step-by-step explanation:

Let suppose that [tex]g(x) = \frac{1}{x^{2}}[/tex], then [tex]f(g(x))[/tex] is:

[tex]f(g(x)) = 7\cdot \left(\frac{1}{x^{2}} \right) + 10[/tex]

[tex]f(g(x)) = 7\cdot g(x) + 10[/tex]

Thus,

[tex]f(x) = 7\cdot x + 10[/tex]

The functions are [tex]f(x) = 7\cdot x+10[/tex] and [tex]g(x) = \frac{1}{x^{2}}[/tex], respectively.

help asap will give 10 points

Answers

Answer:

False

Step-by-step explanation:

[tex]( {9}^{9} ) \times ( {9}^{ - 20} ) \\ = {9}^{9 + ( - 20)} \\ = {9}^{ - 11} [/tex]

Answer:

I'm pretty sure its false

help me im dangered plzzzzzzzzzzzzzzzzzzzz

Answers

Answer:

A

Step-by-step explanation:

Hi!

An exponent is the same thing as just multiplying the expression by itself the number of times the exponent says. So we need to multiply 1/3 by itself three times.

1/3 * 1/3 * 1/3 = 1/27

the answer to your question is A

Joaquin used two types of flour in a muffin recipe. How much flour did he use in all? Solve any way you choose.

Answers

Answer:

3 1/6 cups of flour.

Step-by-step explanation

A circle has a circumference of 153.86153.86153, point, 86 units.
What is the radius of the circle?

Answers

Answer=24.49 I’m not 100% sure but I believe this is the answer :)
Hope it helps

Answer:

Step-by-step explanation:

C=153.86153...

100000C=15386153.86153...

subtract

99999C=15386000

C=15386000/99999

2 π r=15386000/99999

r=7693000/99999π≈24.5

Find the missing probability: P(B)=7/20, P(A|B)=1/4, P(A∩B)=?

Answers

Answer:

P(A∩B) = 7/80

P(A∩B) = 0.0875

Step-by-step explanation:

Given

P(B)=7/20

P(A|B)=¼

Required

P(A∩B)=?

The given probability shows conditional probability and the relationship between the given parameters is as follows.

P(A∩B) = P(B) * P(A|B)

Substitute ¼ for P(A|B) and 7/20 for P(B)

The expression

P(A∩B) = P(B) * P(A|B) becomes

P(A∩B) = 7/20 * ¼

P(A∩B) = 7/80

P(A∩B) = 0.0875

Hence, the calculated P(A∩B) is 7/80 or 0.0875

Name a real world context to describe the sums of rational numbers.

Answers

Step-by-step explanation:

when you are cooking you need to measure fractions of ingredients

What expression has the same value as -3/2-(2-3/8)+3/2

Answers

Answer:

[tex]\dfrac{-3}{2}-(2-\dfrac{3}{8})+\dfrac{3}{2}=\dfrac{-13}{8}[/tex]

Step-by-step explanation:

We need to find the value of expression [tex]\dfrac{-3}{2}-(2-\dfrac{3}{8})+\dfrac{3}{2}[/tex].

Firstly solving the second term as :

[tex](2-\dfrac{3}{8})=\dfrac{16-3}{8}=\dfrac{13}{8}[/tex]

Now the above expression becomes,

[tex]\dfrac{-3}{2}-(2-\dfrac{3}{8})+\dfrac{3}{2}\\=\dfrac{-3}{2}-\dfrac{13}{8}+\dfrac{3}{2}[/tex]

-3/2 and +3/2 equals 0.

It means that, [tex]\dfrac{-3}{2}-(2-\dfrac{3}{8})+\dfrac{3}{2}=\dfrac{-13}{8}[/tex]

Use the Law of Sines to find the missing angle of the triangle.

Find m < C to the nearest tenth if the c= 102, a = 71 and m < A=40

Answers

Answer:

66.9 degrees

Step-by-step explanation:

The Law of Sines states that a/sinA = c/sinC. Plugging in the values for c, a, and M < A, we get:

71/sin40 = 102/sinC

Cross multiplying, we get:

102(sin40deg) = 71(sinC)

Now, we simplify the left side and get:

65.56 = 71(sinC)

Next, we divide 65.56 by 71 to get:

0.92 = sinC

Taking the inverse sign we get:

C = 66.9 degrees

how to find the theta with side lengths of a triangle

Answers

Step-by-step explanation:

Hello, there!!!

I hope you mean the question is like the above problem in picture.

so, let's simply work with it.

here, we may use cosine rule,

so, according to cosine rule,

[tex] {c}^{2} = {a}^{2} + {b}^{2} - 2ab.cosc[/tex]

so, just put value of formulae here,

we get;

5^2 = 3^2 + 4^2 - (2×3×4) . cos thita

or, 25 = 9 + 16 -24 cos thita.

or, 24 cos thita = 0

or, cos thita = 0/25

or, cos thita = 0

now, taking cos to right side we get,

[tex] {cos}^{ - 1 } (0)[/tex]

now, after typing cos ^-1 (0) we get angle as 90°.

(note: in step {cos thita = 0} you couold directly write like; cos thita = cos 90°. and cos would be cancelled in it as cos 90°=0. but it is only applied in particular angle like 0°,30°,60°,..... which are identified or if you don't know you must use the method above using calculator and remember to put inverse {cos^-1}).

so, In this way we find angle.

I hope it helps....

find the area of a rhombus with a 120 degree angle and sides 10 cm

Answers

Answer:

              A = 50√3 cm²

Step-by-step explanation:

[tex]A=s^2\cdot \sin\alpha\\\\s=10\\\alpha=120^o\\\\A=10^2\cdot\sin(120^o)=100\sin(180^o-60^o)=100\sin(60^o)=100\cdot\frac{\sqrt3}2=50\sqrt3[/tex]

What is the area of this composite figure?

Answers

Answer:

C.) 7.14 in²

Step-by-step explanation:

The figure is made up of a square and a circle. The circle is divided in half and each piece is set on one side of the square. This means that the diameter of the circle is equal to the length of the sides of the square, 2 inches.

The area of the square can be found by multiplying length times the width:

[tex]2*2=4[/tex]

The area of the square is 4 inches, and since we multiplied two lengths, we square the value:

A=4in²

Now find the area of the circle using the formula:

[tex]A=\pi r^2[/tex]

The radius is half of the diameter, so the radius is 1. Insert values and solve:

[tex]A=\pi *1^2\\\\A=\pi *1\\\\A=\pi[/tex]

The area of the circle is equal to π. Add the values together:

[tex]4+\pi =7.14[/tex]

The area of the figure is 7.14 in²

:Done

The Muller family are on holiday in New Zealand. a. They change some euros (€) and receive $1962 (New Zealand dollars). The exchange rate is €1 = $1.635. Calculate the number of euros they change. [3] b. The family spend 15% of their New Zealand dollars on a tour. Calculate the number of dollars they have left. [4]

Answers

Answer:

a. €1200;$1667.70

Step-by-step explanation:

a. Number of euros

[tex]\text{euros} = \$1962 \times \dfrac{\text{1 euro}}{\text{\$1.635}} = \textbf{1200 euros}[/tex]

b. Dollars remaining

Dollars on hand                        = $1962.00

Less 15 % spent = 0.15 × 1962 =   -294.30

Balance remaining                   =  $1667.70

PLEASE HELP Ruri is a 30-year-old math teacher. She has been informed that she is the winner of a grand prize for the lottery. She can choose either a one-time payment of $20 million or $5000 per week for the rest of her life. Which choice would most likely result in the greatest amount of winnings for Ruri? Explain your reasoning.

Answers

Answer:

$5,000 per week

Step-by-step explanation:

Ruri is a 30 year old female.

there are about 4 weeks per month

there are about 52 weeks per year

52*5000 = 260,000

She would get 260,000 per year and lets see how much she would have at 40.

260,000*10

at 40 she would have 2,600,000

2,600,000*10

at 50 she would have 26,000,000

at 50 she already has earned more money that the $20 million.

She should go with the $5000 per week if she would like more money.

HELP ASAP ITS SO HARD! Kelsey did the following division problem. Her teacher says that the quotient she found is wrong. −2 5/6 ÷ 1 1/3 −17/6 ÷ 4/3 −6/17• 3/4 −6×3 divided by 17×4 −18/68 −9/34 A. Identify what Kelsey did wrong in her calculations. B. Find the correct quotient, showing all of your calculations.

Answers

Part A

Her steps were

[tex]-2 \frac{5}{6} \div 1 \frac{1}{3}\\\\-\frac{17}{6} \div \frac{4}{3}\\\\-\frac{6}{17} \times \frac{3}{4}\\\\-\frac{6\times 3}{17\times4}\\\\-\frac{18}{68}\\\\-\frac{9}{34}\\\\[/tex]

Kelsey made a mistake on line 3. Note how the 17/6 flips to 6/17. This is not correct. You keep the first fraction the same, but you do flip the second fraction. This only applies when you divide two fractions.

The third step should look like [tex]-\frac{17}{6}\times \frac{3}{4}[/tex]

=======================================================

Part B

Here's what she should have written

[tex]-2 \frac{5}{6} \div 1 \frac{1}{3}\\\\-\frac{17}{6} \div \frac{4}{3}\\\\-\frac{17}{6} \times \frac{3}{4}\\\\-\frac{17\times 3}{6\times 4}\\\\-\frac{51}{24}\\\\-\frac{17}{8}\\\\[/tex]

If you want to convert that improper fraction to a mixed number, then you could do something like this

[tex]-\frac{17}{8} = -\frac{16+1}{8}\\\\-\frac{17}{8} = -\frac{16}{8}-\frac{1}{8}\\\\-\frac{17}{8} = -2 \frac{1}{8}\\\\[/tex]

Or you could divide 17 over 8 using long division to get 2 remainder 1. The 2 is the quotient that goes to the left of the 1/8. The remainder of 1 is the numerator of 1/8.

A can do a piece of work in 10 days and B in 12 days.
With the help of C they can finish the work in 4 days. How
long will C take to do it alone?
a) 12 days b) 15 days c) 18 days d) 20 days​

Answers

Answer:

B. 15 days

Step-by-step explanation:

A=10=1/10 of the job

B=12=1/12 of the job

A+B+C=4=1/4 of the job

A+B

=1/10 + 1/12

=10+12/120

=22/120

=11/60

(A+B+C) - (A+B)

=1/4 - 11/60

=5 - 11/60

=4/60

=1/15

It would take C alone 15 days to finish the job

need help please. Will give you 5-stars and a big thank you comrades

Answers

Answer:

first answer

Step-by-step explanation:

(8x³ - 22x² - 4) / (4x - 3)

when you do long division you get the first answer

solve 2(1/9)× = 2/81 for x​

Answers

Answer: x=1/9

Step-by-step explanation:

[tex]2\left(\frac{1}{9}\right)x=\frac{2}{81}[/tex]

[tex]\frac{2}{9}x=\frac{2}{81}[/tex]

multiply both sides by 9

[tex]9\cdot \frac{2}{9}x=\frac{2\cdot \:9}{81}[/tex]

[tex]2x=\frac{2}{9}[/tex]

divide 2 on both sides

[tex]x=\frac{1}{9}[/tex]

Below are a list of costs and discounts for groceries. Round to the nearest dollar to estimate the total cost
+ $12.34
+ $5.07
- $0.73
+ $2.84
- $1.50

Answers

Answer:

$18

Step-by-step explanation:

$18.02 rounds to $18

Answer:

  $17

Step-by-step explanation:

Rounded, the listed numbers are ...

  12 + 5 -1 +3 -2 = 17

The estimate of total cost is $17.

find the midpoint of the line segment whose endpoints are
(5,9) (2,-1)?​

Answers

Answer:

(3.5,4)

Step by step explanation:

x coordinate =

[tex] \frac{5 + 2}{2} [/tex]

= 3.5

y coordinate =

[tex] \frac{ - 1 + 9}{2} [/tex]

= 4

midpoint = (3.5,4)

Answer:

(  3.5,4)

Step-by-step explanation:

To find the midpoint

Add the x coordinates together and divide by 2

(5+2)/2 = 7/2 = 3.5

Add the y coordinates together and divide by 2

(9+-1)/2 = 8/2 = 4

( 3.5,4)

I also need to know this one pretty soon pleaseee​

Answers

Answer:

128

Step-by-step explanation:

what is the answer for 6x-4=-26+5x

Answers

Answer: x=-22

Step-by-step explanation:

    6x-4=-26+5x

6x-4-5x=-26+5x-5x ⇔ subtraction property of equality

       x-4=-26

  x-4+4=-26+4 ⇔ addition property of equality

         x=-22

Answer:

x = - 22

Step-by-step explanation:

6x - 4 = - 26 + 5x

First of all group like terms

Send the constants to the right side of the equation and those with variables to the left

That's

6x - 5x = 4 - 26

Simplify

We have the final answer as

x = - 22

Hope this helps you

stagg high school has a rectangular swiming pool the area of the water in the pool is 1,800 meters squared the length is twice the width what is the perimeter of the pool find the length and width. SHOW WORK

Answers

Step-by-step explanation:

L*b=1800m^2

L=2b

2b*b=1800

2b^2=1800

b^2=900

b=30m

L=2*30

=60m

Perimeter=2(l+b)

=2(60+30)

=2*90

=180m

help help help me plZZZZZ ill give you brainly ;DDD

Answers

Answer:

the answer is 60.7

Step-by-step explanation:

60 to has a between numbers like given in the picture

so as number line it's

60.1 . 60.2 60.3 60.4 60.5 60.6 60.7 60.8 and continue

if u get any 3 digit number like 600 to 650 in number line

u do it like it the same 600.1 600.2.... and go on

Answer:

63½ or 63.5

Step-by-step explanation:

65-60=5

10points=5

1point=?

1×5/10= ½

that means the sequence continues after adding ½ i.e

60..60½...61...61½...62...62½...63...63½...64..64½...65

you have been asked the 8th number which is 63½

Please help At a party of 102 people, 72 lb of potato salad is served. Write and solve and equation to find how many people each pound of potato salad serves.

Answers

Answer:

1.417

Step-by-step explanation:

102/72=1.417

Ten people were chosen at random and surveyed. The survey asked participants for the number of hours they sleep per night and the amount of their annual income. Letting X represent the number of hours the participant sleeps per night and Y represent the participant's annual income, the surveyor calculated the correlation coefficient between X and Y to be 0.29. Interpret the correlation coefficient calculated by choosing the statement below which correctly describes the correlation between X and Y. A. weak negative correlation B. strong negative correlation C. strong positive correlation D. weak positive correlation

Answers

Answer:

A. R=0.86; strong correlation

Step-by-step explanation:

The correlation coefficient of 0.29 indicates that the correlation is a weak positive correlation. Thus option (D) is the correct answer.

What is correlation?

"Correlation is a statistical tool that studies the relationship between two variables. Data sets have a positive correlation when they increase together, and a negative correlation when one set increases as the other decreases".

For the given situation,

Correlation coefficient = 0.29

Positive correlation: the two variables change in the same direction.

Negative correlation: the two variables change in opposite directions.

No correlation: there is no association or relevant relationship between the two variables.

The correlation coefficient lies between 0 to 0.3 indicating that the correlation is a weak positive correlation.

Hence we can conclude that option (D) weak positive correlation is the correct answer.

Learn more about correlation here

https://brainly.com/question/10721912

#SPJ2

Other Questions
How were the Olive Branch Petition and Common-Sense representative of the Loyalist and Patriot points of view? A manufacturing company is expected to pay a dividend of br. 1.25 per share at the end of the year (D1=br.1.25). The stock sells for br. 32.50 per share and its required rate of return is 10.5%. The dividend is expected to grow at some constant rate forever. What is the growth rate PLEASE HELPPPPP!!!!!!!!!!!!!!!Which relationships have the same constant of proportionality between y and x as the following graph?Choose two answers!! FIFO Perpetual Inventory The beginning inventory at Dunne Co. and data on purchases and sales for a three-month period ending June 30 are as follows: Date Transaction Number of Units Per Unit Total Apr. 3 Inventory 48 $150 $7,200 8 Purchase 96 180 17,280 11 Sale 64 500 32,000 30 Sale 40 500 20,000 May 8 Purchase 80 200 16,000 10 Sale 48 500 24,000 19 Sale 24 500 12,000 28 Purchase 80 220 17,600 June 5 Sale 48 525 25,200 16 Sale 64 525 33,600 21 Purchase 144 240 34,560 28 Sale 72 525 37,800 Required: 1. Record the inventory, purchases, and cost of merchandise sold data in a perpetual inventory record similar to the one illustrated in Exhibit 3, using the first-in, first-out method. Under FIFO, if units are in inventory at two different costs, enter the units with the LOWER unit cost first in the Cost of Goods Sold Unit Cost column and in the Inventory Unit Cost column. Dunne Co. Schedule of Cost of Goods Sold FIFO Method For the Three Months Ended June 30 Purchases Cost of Goods Sold Inventory Date Quantity Unit Cost Total Cost Quantity Unit Cost Total Cost Quantity Unit Cost Total Cost Apr. 3 $ $ Apr. 8 $ $ Apr. 11 $ $ Apr. 30 May 8 May 10 May 19 May 28 June 5 June 16 June 21 June 28 June 30 Balances $ $ 2. Determine the total sales and the total cost of goods sold for the period. Journalize the entries in the sales and cost of goods sold accounts. Assume that all sales were on account. Record sale Record cost 3. Determine the gross profit from sales for the period. $ 4. Determine the ending inventory cost as of June 30. $ 5. Based upon the preceding data, would you expect the ending inventory using the last-in, first-out method to be higher or lower? In the following sentence, which word(s) signifies order in the paragraph? (Secondh; you must add salt and pepper to the avocado dressing,O andO SecondlyOaddmust What is the value of 1 in 1,255 is what times the value of the 1 in 82,175 A hot metal plate at 150C has been placed in air at room temperature. Which event would most likely take placeover the next few minutes?Molecules in both the metal and the surrounding air will start moving at lower speeds.Molecules in both the metal and the surrounding air will start moving at higher speeds.The air molecules that are surrounding the metal will slow down, and the molecules in the metal will speed upThe air molecules that are surrounding the metal will speed up, and the molecules in the metal will slow down Discount stores that try to keep prices as low as possible are more likely to function using ________ operations. Burpee Company sells seeds to garden stores. Sales are expected to be $2,038,635 in January, $2,581,891 in February and $2,913,307 in March. Burpee sets their prices so that they earn an average 32% gross profit on sales revenue. What is budgeted cost of goods sold for the first quarter (January, February and March)? Do students prefer public school or homeschool during the pandemic? the dot plot above identifies the number of pets living with each of 20 families in an apartment building .what fraction of families have more than two pets what is acceleration produced by a force of 12 newton exerted on an object of mass 3kg do numbers ever stop Managers should be more concerned that data is stored in a structured data warehouse or a Hadoop cluster, and less about the actually the insights that can be derived from the data. True False A simple random sample of 20 observations is derived from a normally distributed population with a known standard deviation of 3.2. You may find it useful to reference the z table.a. Is the condition that XX is normally distributed satisfied?YesNob. Compute the margin of error with 95% confidence. (Round intermediate calculations to at least 4 decimal places. Round "z" value to 3 decimal places and final answer to 2 decimal places.)c. Compute the margin of error with 90% confidence. (Round intermediate calculations to at least 4 decimal places. Round "z" value to 3 decimal places and final answer to 2 decimal places.)d. Which of the two margins of error will lead to a wider interval?The margin of error with 95% confidence.The margin of error with 90% confidence. What is the formula of a compound if a sample of the compound contains 0.492 mol X, 0.197 mol Y, and 0.295 mol Z? At Polar Sportswear, orders have significantly exceeded projections, and Chris, the operations director, has decided to hire for a third shift in the plant. Chris is clearly operating with the use of a(n) ______ budget. Suppose that Mexico experienced a very severe period of inflation in 1972. As prices in Mexico rose, the demand in the foreign exchange market for Mexican pesos: HELP SOMEONE PLEASE!!!!! Factor completely 10x2 + 2x 8. 2(5x 1)(x + 4) 2(5x 4)(x + 1) 2(5x + 2)(x 2) 2(5x 2)(x + 2) the length of rectangle is 6/5 of its breath and perimeter is 132 m find area of rectangle